Mini GMAT Quant. Test - 10 Questions

Approved & Edited by ProProfs Editorial Team
The editorial team at ProProfs Quizzes consists of a select group of subject experts, trivia writers, and quiz masters who have authored over 10,000 quizzes taken by more than 100 million users. This team includes our in-house seasoned quiz moderators and subject matter experts. Our editorial experts, spread across the world, are rigorously trained using our comprehensive guidelines to ensure that you receive the highest quality quizzes.
Learn about Our Editorial Process
| By Dakarazu
D
Dakarazu
Community Contributor
Quizzes Created: 4 | Total Attempts: 6,863
Questions: 79 | Attempts: 6,037

SettingsSettingsSettings
Mini GMAT Quant. Test - 10 Questions - Quiz

This is a GMAT quantitative diagnostic test: Exam 1.


Questions and Answers
  • 1. 

    When tossed, a certain coin has an equal chance of landing on either side.  If the coin is tossed 4 times, what is the probability that it will land on the same side each time?  

    Correct Answer
    D.
  • 2. 

    There are 6 pairs of balls with each pair in a different color. If we select 2 from the 12 balls without replacement, what’s the probability of selecting 2 balls with the same color?  

    Correct Answer
    E.
    Explanation
    The probability of selecting 2 balls with the same color can be calculated by dividing the number of favorable outcomes (selecting 2 balls of the same color) by the total number of possible outcomes. Since there are 6 pairs of balls in different colors, there are 6 different colors to choose from. The first ball can be any of the 12, and the second ball must be of the same color as the first. Therefore, there are 12 possible choices for the first ball and only 1 choice for the second ball. The total number of possible outcomes is 12 * 11. Thus, the probability is 1/66.

    Rate this question:

  • 3. 

    A glass was filled with 10 ounces of water, and 0.01 ounce of the water evaporated each day during a twenty-day period.  What percent of the original amount of water evaporated during this period?  

    • A.

      0.002%

    • B.

      0.02%

    • C.

      0.2%

    • D.

      2%

    • E.

      20%

    Correct Answer
    D. 2%
    Explanation
    During the twenty-day period, 0.01 ounce of water evaporated each day. To find the total amount of water evaporated, we multiply 0.01 ounce by 20 days, which equals 0.2 ounces. To find the percentage, we divide the amount evaporated (0.2 ounces) by the original amount of water (10 ounces) and multiply by 100. This gives us a percentage of 2%.

    Rate this question:

  • 4. 

    If x and y are integers and x > 0, is y > 0?   (1)   7x – 2y > 0   (2)   -y < x  

    • A.

      Statement (1) ALONE is sufficient, but statement (2) alone is not sufficient.

    • B.

      Statement (2) ALONE is sufficient, but statement (1) alone is not sufficient.

    • C.

      BOTH statements TOGETHER are sufficient, but NEITHER statement ALONE is sufficient.

    • D.

      EACH statement ALONE is sufficient.

    • E.

      Statements (1) and (2) TOGETHER are NOT sufficient.

    Correct Answer
    E. Statements (1) and (2) TOGETHER are NOT sufficient.
    Explanation
    To determine if y is greater than 0, we need to analyze both statements together.

    From statement (1), we have 7x - 2y > 0. This means that 7x > 2y. However, we do not have any information about the relation between x and y, so we cannot determine if y is greater than 0 based on this statement alone.

    From statement (2), we have -y < x. This means that y > -x. Again, we do not have enough information to determine if y is greater than 0 based on this statement alone.

    Since neither statement alone provides enough information, and we need both statements together to determine if y is greater than 0, the correct answer is that statements (1) and (2) together are not sufficient.

    Rate this question:

  • 5. 

    Each week a certain salesman is made a fixed amount equal to $300 plus a commission equal to 5 percent of the amount of these sales that week over $1,000.  What is the total amount the salesman was paid last week?   (1)   The total amount the salesman was paid last week is equal to 10 percent of the amount of these sales last week.   (2)   The salesman’s sales last week total $5,000.  

    • A.

      Statement (1) ALONE is sufficient, but statement (2) alone is not sufficient.

    • B.

      Statement (2) ALONE is sufficient, but statement (1) alone is not sufficient.

    • C.

      BOTH statements TOGETHER are sufficient, but NEITHER statement ALONE is sufficient.

    • D.

      EACH statement ALONE is sufficient.

    • E.

      Statements (1) and (2) TOGETHER are NOT sufficient.

    Correct Answer
    D. EACH statement ALONE is sufficient.
    Explanation
    The correct answer is EACH statement ALONE is sufficient.



    Statement (1) provides the information that the total amount the salesman was paid last week is equal to 10 percent of the amount of these sales last week. This allows us to calculate the total amount the salesman was paid last week based on the sales amount.

    Statement (2) provides the information that the salesman's sales last week total $5,000. This gives us the sales amount needed to calculate the total amount the salesman was paid last week.

    Therefore, each statement alone provides enough information to calculate the total amount the salesman was paid last week.

    Rate this question:

  • 6. 

    If S is a set of ten consecutive integers, is the integer 5 in S? (1)   The integer –3 is in S. (2)   The integer 4 is in S.  

    • A.

      Statement (1) ALONE is sufficient, but statement (2) alone is not sufficient.

    • B.

      Statement (2) ALONE is sufficient, but statement (1) alone is not sufficient.

    • C.

      BOTH statements TOGETHER are sufficient, but NEITHER statement ALONE is sufficient.

    • D.

      EACH statement ALONE is sufficient.

    • E.

      Statements (1) and (2) TOGETHER are NOT sufficient.

    Correct Answer
    E. Statements (1) and (2) TOGETHER are NOT sufficient.
    Explanation
    From statement (1), we know that the integer -3 is in S. Since S is a set of ten consecutive integers, we can determine that the lowest integer in S is -3 and the highest integer in S is 6. However, we still don't have enough information to determine if the integer 5 is in S.

    From statement (2), we know that the integer 4 is in S. This gives us some information about the range of S, but it still doesn't tell us if the integer 5 is in S.

    By combining both statements, we know that the lowest integer in S is -3 and the highest integer in S is 6, and that the integer 4 is in S. However, we still don't have enough information to determine if the integer 5 is in S. Therefore, both statements together are not sufficient to answer the question.

    Rate this question:

  • 7. 

    Is xy > x2y2? (1)   14x2 = 3 (2)    y2 = 1  

    • A.

      Statement (1) ALONE is sufficient, but statement (2) alone is not sufficient.

    • B.

      Statement (2) ALONE is sufficient, but statement (1) alone is not sufficient.

    • C.

      BOTH statements TOGETHER are sufficient, but NEITHER statement ALONE is sufficient.

    • D.

      EACH statement ALONE is sufficient.

    • E.

      Statements (1) and (2) TOGETHER are NOT sufficient.

    Correct Answer
    E. Statements (1) and (2) TOGETHER are NOT sufficient.
    Explanation
    The question asks whether xy > x^2y^2.

    Statement (1) tells us that 14x^2 = 3. However, this does not provide any information about the value of y, so we cannot determine the relationship between xy and x^2y^2 based on this statement alone.

    Statement (2) tells us that y^2 = 1. This means that y could be either 1 or -1. Again, we do not have enough information to determine the relationship between xy and x^2y^2 based on this statement alone.

    When we consider both statements together, we still cannot determine the relationship between xy and x^2y^2. Even though we know the value of x^2y^2 from statement (1) and the value of y from statement (2), we do not have enough information to determine the relationship between xy and x^2y^2.

    Therefore, the answer is that statements (1) and (2) together are not sufficient to answer the question.

    Rate this question:

  • 8. 

    Is x2 + y2 > 6? (1)   (x + y)2 > 6 (2)    xy = 2  

    • A.

      Statement (1) ALONE is sufficient, but statement (2) alone is not sufficient.

    • B.

      Statement (2) ALONE is sufficient, but statement (1) alone is not sufficient.

    • C.

      BOTH statements TOGETHER are sufficient, but NEITHER statement ALONE is sufficient.

    • D.

      EACH statement ALONE is sufficient.

    • E.

      Statements (1) and (2) TOGETHER are NOT sufficient.

    Correct Answer
    E. Statements (1) and (2) TOGETHER are NOT sufficient.
    Explanation
    From statement (1), we know that (x + y)^2 > 6. However, we cannot determine if x^2 + y^2 > 6 or x^2 + y^2 < 6. Therefore, statement (1) alone is not sufficient to answer the question.

    From statement (2), we know that xy = 2. However, we cannot determine the values of x and y individually or their sum. Therefore, statement (2) alone is not sufficient to answer the question.

    By combining both statements, we still cannot determine if x^2 + y^2 > 6 or x^2 + y^2 < 6. Therefore, statements (1) and (2) together are not sufficient to answer the question.

    Rate this question:

  • 9. 

    If s4v3x7 < 0, is svx < 0? (1)   v < 0 (2)   x > 0  

    • A.

      Statement (1) ALONE is sufficient, but statement (2) alone is not sufficient.

    • B.

      Statement (2) ALONE is sufficient, but statement (1) alone is not sufficient.

    • C.

      BOTH statements TOGETHER are sufficient, but NEITHER statement ALONE is sufficient.

    • D.

      EACH statement ALONE is sufficient.

    • E.

      Statements (1) and (2) TOGETHER are NOT sufficient.

    Correct Answer
    E. Statements (1) and (2) TOGETHER are NOT sufficient.
    Explanation
    From statement (1), we know that v < 0. However, this does not provide any information about the value of x. Therefore, we cannot determine if svx < 0 based on this statement alone.

    From statement (2), we know that x > 0. However, this does not provide any information about the value of v. Therefore, we cannot determine if svx < 0 based on this statement alone.

    Since neither statement alone provides enough information to determine if svx < 0, we need to consider both statements together. However, even when considering both statements together, we still do not have enough information to determine if svx < 0. Therefore, the answer is that statements (1) and (2) together are not sufficient.

    Rate this question:

  • 10. 

    Is += 1 ? (1)   x  0 (2)   x  1  

    • A.

      Statement (1) ALONE is sufficient, but statement (2) alone is not sufficient.

    • B.

      Statement (2) ALONE is sufficient, but statement (1) alone is not sufficient.

    • C.

      BOTH statements TOGETHER are sufficient, but NEITHER statement ALONE is sufficient.

    • D.

      EACH statement ALONE is sufficient.

    • E.

      Statements (1) and (2) TOGETHER are NOT sufficient.

    Correct Answer
    C. BOTH statements TOGETHER are sufficient, but NEITHER statement ALONE is sufficient.
    Explanation
    This answer suggests that both statement (1) and statement (2) together are sufficient to determine whether += 1 is true or not. However, neither statement alone is sufficient to make this determination.

    Rate this question:

  • 11. 

    Is is  > 0 ? (1)   (2)    

    • A.

      Statement (1) ALONE is sufficient, but statement (2) alone is not sufficient.

    • B.

      Statement (2) ALONE is sufficient, but statement (1) alone is not sufficient.

    • C.

      BOTH statements TOGETHER are sufficient, but NEITHER statement ALONE is sufficient.

    • D.

      EACH statement ALONE is sufficient.

    • E.

      Statements (1) and (2) TOGETHER are NOT sufficient.

    Correct Answer
    A. Statement (1) ALONE is sufficient, but statement (2) alone is not sufficient.
    Explanation
    Since there is no information provided in either statement (1) or statement (2), we cannot determine the value of "is". Therefore, statement (1) alone is not sufficient to answer the question. However, since statement (2) also does not provide any information, it is also not sufficient to answer the question. Therefore, the correct answer is that neither statement alone is sufficient, but statement (1) alone is sufficient.

    Rate this question:

  • 12. 

    Each of the 45 boxes on shelf J weighs less than each of the 44 boxes on shelf K.  What is the median weight of the 89 boxes on these shelves? (1)   The heaviest box on shelf J weighs 15 pounds. (2)    The lightest box on shelf K weighs 20 pounds.  

    • A.

      Statement (1) ALONE is sufficient, but statement (2) alone is not sufficient.

    • B.

      Statement (2) ALONE is sufficient, but statement (1) alone is not sufficient.

    • C.

      BOTH statements TOGETHER are sufficient, but NEITHER statement ALONE is sufficient.

    • D.

      EACH statement ALONE is sufficient.

    • E.

      Statements (1) and (2) TOGETHER are NOT sufficient.

    Correct Answer
    A. Statement (1) ALONE is sufficient, but statement (2) alone is not sufficient.
    Explanation
    Statement (1) alone is sufficient to determine the median weight of the boxes. Since the heaviest box on shelf J weighs 15 pounds and each of the 45 boxes on shelf J weighs less than each of the 44 boxes on shelf K, it can be concluded that the median weight of the boxes on shelf J is less than 15 pounds.

    Statement (2) alone does not provide any information about the weights of the boxes on shelf J or the relationship between the weights of the boxes on shelf J and shelf K. Therefore, it is not sufficient to determine the median weight of the boxes.

    Hence, statement (1) alone is sufficient to determine the median weight of the 89 boxes on these shelves.

    Rate this question:

  • 13. 

    What is the total value of Company H&rsquos stock?     (1)   Investor P owns  of the shares of Company H&rsquos total stock.   (2)   The total value of Investor Q&rsquos shares of Company H&rsquos stock is $16000.  

    • A.

      Statement (1) ALONE is sufficient, but statement (2) alone is not sufficient.

    • B.

      Statement (2) ALONE is sufficient, but statement (1) alone is not sufficient.

    • C.

      BOTH statements TOGETHER are sufficient, but NEITHER statement ALONE is sufficient.

    • D.

      EACH statement ALONE is sufficient.

    • E.

      Statements (1) and (2) TOGETHER are NOT sufficient.

    Correct Answer
    E. Statements (1) and (2) TOGETHER are NOT sufficient.
    Explanation
    The total value of Company H's stock cannot be determined based on the given statements alone. Statement (1) only provides information about the ownership of the shares by Investor P, but it does not give any information about the total value of the stock. Statement (2) only provides information about the total value of Investor Q's shares, but it does not give any information about the total value of the stock or the ownership of other investors. Therefore, both statements together are not sufficient to determine the total value of Company H's stock.

    Rate this question:

  • 14. 

    If   what is the value of ?   (1)       (2)      

    • A.

      Statement (1) ALONE is sufficient, but statement (2) alone is not sufficient.

    • B.

      Statement (2) ALONE is sufficient, but statement (1) alone is not sufficient.

    • C.

      BOTH statements TOGETHER are sufficient, but NEITHER statement ALONE is sufficient.

    • D.

      EACH statement ALONE is sufficient.

    • E.

      Statements (1) and (2) TOGETHER are NOT sufficient.

    Correct Answer
    B. Statement (2) ALONE is sufficient, but statement (1) alone is not sufficient.
  • 15. 

    If the length of a certain rectangle is 2 greater than the width of the rectangle, what is the perimeter of the rectangle?   (1)   The length of each diagonal of the rectangle is 10.   (2)   The area of the rectangular region is 48.  

    • A.

      Statement (1) ALONE is sufficient, but statement (2) alone is not sufficient.

    • B.

      Statement (2) ALONE is sufficient, but statement (1) alone is not sufficient.

    • C.

      BOTH statements TOGETHER are sufficient, but NEITHER statement ALONE is sufficient.

    • D.

      EACH statement ALONE is sufficient.

    • E.

      Statements (1) and (2) TOGETHER are NOT sufficient.

    Correct Answer
    D. EACH statement ALONE is sufficient.
    Explanation
    Statement (1) alone is sufficient to find the perimeter of the rectangle. If the length of each diagonal is 10, we can use the Pythagorean theorem to find the relationship between the length and width of the rectangle. Since the length is 2 greater than the width, we can set up the equation x^2 + (x+2)^2 = 10^2, where x is the width. Solving this equation will give us the width, and we can then calculate the perimeter using the formula P = 2(length + width). Therefore, statement (1) alone is sufficient to find the perimeter.

    Rate this question:

  • 16. 

    If n and k are positive integers is  an even integer?   (1)   n is divisible by 8.   (2)   k is divisible by 4.  

    • A.

      Statement (1) ALONE is sufficient, but statement (2) alone is not sufficient.

    • B.

      Statement (2) ALONE is sufficient, but statement (1) alone is not sufficient.

    • C.

      BOTH statements TOGETHER are sufficient, but NEITHER statement ALONE is sufficient.

    • D.

      EACH statement ALONE is sufficient.

    • E.

      Statements (1) and (2) TOGETHER are NOT sufficient.

    Correct Answer
    E. Statements (1) and (2) TOGETHER are NOT sufficient.
    Explanation
    Statement (1) alone tells us that n is divisible by 8. However, this does not guarantee that n is even. For example, n could be 16, which is even, or it could be 24, which is not even. Therefore, statement (1) alone is not sufficient to determine if n is an even integer.

    Statement (2) alone tells us that k is divisible by 4. This does not provide any information about n, so it is not sufficient to determine if n is an even integer.

    When we consider both statements together, we still do not have enough information to determine if n is an even integer. The statements provide information about n and k separately, but do not provide any relationship between them. Therefore, statements (1) and (2) together are not sufficient to determine if n is an even integer.

    Rate this question:

  • 17. 

    If M is the least common multiple of 90, 196, and 300, which of the following is NOT a factor of M?  

    • A.

      600

    • B.

      700

    • C.

      900

    • D.

      2,100

    • E.

      4,900

    Correct Answer
    A. 600
    Explanation
    To find the least common multiple (LCM) of 90, 196, and 300, we need to find the smallest number that is divisible by all three numbers. The LCM of these numbers is 2,100. Therefore, all the given options are factors of the LCM except for 600. Hence, 600 is NOT a factor of the LCM.

    Rate this question:

  • 18. 

    In a survey of 200 college graduates, 30 percent said they had received student loans during their college careers, and 40 percent said they had received scholarships.  What percent of those surveyed said that they had received neither student loans nor scholarships during their college careers? (1)   25 percent of those surveyed said that they had received scholarships but no loans. (2)   50 percent of those surveyed who said that they had received loans also said that they had received scholarships.  

    • A.

      Statement (1) ALONE is sufficient, but statement (2) alone is not sufficient.

    • B.

      Statement (2) ALONE is sufficient, but statement (1) alone is not sufficient.

    • C.

      BOTH statements TOGETHER are sufficient, but NEITHER statement ALONE is sufficient.

    • D.

      EACH statement ALONE is sufficient.

    • E.

      Statements (1) and (2) TOGETHER are NOT sufficient.

    Correct Answer
    D. EACH statement ALONE is sufficient.
    Explanation
    Statement (1) alone is sufficient to determine the percent of those surveyed who said they had received neither student loans nor scholarships. It states that 25 percent of those surveyed said they had received scholarships but no loans. Since 30 percent said they had received student loans and 40 percent said they had received scholarships, we can subtract these percentages from 100 to find the percentage of those who received neither. Therefore, the percent of those surveyed who said they received neither student loans nor scholarships is 100 - 30 - 40 - 25 = 5 percent.

    Similarly, statement (2) alone is also sufficient to determine the percent of those surveyed who said they had received neither student loans nor scholarships. It states that 50 percent of those surveyed who said they had received loans also said they had received scholarships. Since 30 percent said they had received student loans, we can subtract this percentage from 100 to find the percentage of those who received neither. Therefore, the percent of those surveyed who said they received neither student loans nor scholarships is 100 - 30 = 70 percent.

    Hence, each statement alone is sufficient to determine the percent of those surveyed who said they had received neither student loans nor scholarships.

    Rate this question:

  • 19. 

    In a poll involving two yes- or no-questions, all the people who were asked the two questions answered “yes” or “no”. If 75 percent of these people answered “yes” to the first question, 55 percent of the people answered “yes” to the second question, and 20 percent of the people answered “no” to both questions, what percentage of the people answered “yes” to both questions?   

    • A.

      70%

    • B.

      50%

    • C.

      25%

    • D.

      20%

    • E.

      10%

    Correct Answer
    B. 50%
  • 20. 

    One day Hotel Merinda rented 75 percent of its rooms, including  of its luxury rooms. If 60 percent of Hotel Merinda’s rooms are luxury rooms, what percent of the rooms that were NOT rented are luxury rooms?  

    • A.

      80%

    • B.

      40%

    • C.

      30%

    • D.

      33.33%

    • E.

      10%

    Correct Answer
    A. 80%
    Explanation
    Since 60% of the hotel's rooms are luxury rooms, it means that 40% of the rooms are not luxury rooms.
    If 75% of the hotel's rooms were rented, it means that 25% of the rooms were not rented.
    Out of those 25% of rooms that were not rented, 40% of them are luxury rooms.
    Therefore, the percentage of the rooms that were not rented and are luxury rooms is 40% of 25%, which is equal to 10%.
    However, the question is asking for the percentage of the rooms that were not rented and are luxury rooms, so we need to find the complement of 10%, which is 100% - 10% = 90%.
    Therefore, the correct answer is 90%.

    Rate this question:

  • 21. 

    2x + y = 12                                      &le 12 For how many ordered pairs (x y) that are solutions of the system above are x and y both integers?  

    • A.

      7

    • B.

      10

    • C.

      12

    • D.

      13

    • E.

      14

    Correct Answer
    D. 13
    Explanation
    The system of equations is 2x + y = 12 and x + y ≤ 12. To find the number of ordered pairs (x, y) that are solutions to this system where both x and y are integers, we can analyze the possible values of x and y. Since x and y are integers, we can start by finding the range of values for x and y that satisfy the given conditions. From the equation 2x + y = 12, we know that y must be an even number for 2x + y to equal 12. Therefore, the possible values for y are 2, 4, 6, 8, 10, and 12. For each value of y, we can solve the equation 2x + y = 12 to find the corresponding value of x. By doing this, we find that there are 13 ordered pairs (x, y) that are solutions to the system where both x and y are integers.

    Rate this question:

  • 22. 

    Mary walked from home to work in the morning at an average speed of 3 miles per hour. She returned in the evening taking the same route running an average speed of 6 miles per hour. What was her average speed, in miles per hour, for the whole journey to and from work?   

    • A.

      3.5

    • B.

      4

    • C.

      4.5

    • D.

      5

    • E.

      5.5

    Correct Answer
    B. 4
    Explanation
    Mary's average speed for the whole journey can be calculated by taking the total distance traveled and dividing it by the total time taken. Since Mary traveled the same route both ways, the total distance is the same for the morning and evening trips. Let's assume the distance is D miles.

    In the morning, Mary traveled at a speed of 3 miles per hour, so the time taken for the morning trip is D/3 hours.

    In the evening, Mary traveled at a speed of 6 miles per hour, so the time taken for the evening trip is D/6 hours.

    The total time taken for the whole journey is D/3 + D/6 = (2D + D)/6 = 3D/6 = D/2 hours.

    Therefore, the average speed for the whole journey is D miles / (D/2 hours) = 2 miles per hour.

    Hence, the correct answer is 2, which is not listed as an option.

    Rate this question:

  • 23. 

    Pierre is driving 50 miles per hour for the first 20 miles of a 40-mile trip. What must be his average speed in the remaining 20 miles in order for his total average speed to be 60 miles per hour?  

    • A.

      60

    • B.

      65

    • C.

      70

    • D.

      75

    • E.

      80

    Correct Answer
    D. 75
    Explanation
    To find the average speed for the entire trip, we can use the formula: average speed = total distance / total time. We know that the total distance is 40 miles and the total average speed is 60 miles per hour. Let's assume that Pierre drives at an average speed of x miles per hour for the remaining 20 miles. We can set up the equation: 60 = 40 / (20/50 + 20/x). Simplifying this equation, we get: 60 = 40 / (2/5 + 20/x). Cross-multiplying, we get: 60 * (2/5 + 20/x) = 40. Simplifying further, we get: 2/5 + 20/x = 2/3. Solving for x, we find that x = 75. Therefore, Pierre must drive at an average speed of 75 miles per hour for the remaining 20 miles in order for his total average speed to be 60 miles per hour.

    Rate this question:

  • 24. 

    In how many different ways can a chairperson and a secretary be selected from a committee of 9 people?  

    • A.

      24

    • B.

      36

    • C.

      72

    • D.

      80

    • E.

      120

    Correct Answer
    C. 72
    Explanation
    In order to select a chairperson and a secretary from a committee of 9 people, we need to consider the order in which they are selected. The chairperson can be selected in 9 different ways (as there are 9 people to choose from), and once the chairperson is selected, the secretary can be selected in 8 different ways (as there are now only 8 people left to choose from). Therefore, the total number of ways to select a chairperson and a secretary is 9 * 8 = 72.

    Rate this question:

  • 25. 

    On Saturday morning, Malachi will begin a camping vacation and he will return home at the end of the first day on which it rains.  If on the first three days of the vacation the probability of rain on each day is 0.2, what is the probability that Malachi will return home at the end of the day on the following Monday?  

    • A.

      0.008

    • B.

      0.128

    • C.

      0.488

    • D.

      0.512

    • E.

      0.640

    Correct Answer
    B. 0.128
    Explanation
    The probability of Malachi returning home at the end of the first day it rains is 0.2. Since it is given that Malachi will return home at the end of the first day it rains, the probability of it raining on the first day is 0.2. Therefore, the probability of it not raining on the first day is 1 - 0.2 = 0.8. Since it is not specified when the camping vacation starts, we assume it starts on Saturday. Therefore, the probability of it not raining on Saturday, Sunday, and Monday is (0.8)^3 = 0.512. Therefore, the probability of Malachi returning home at the end of the day on Monday is 0.512 * 0.2 = 0.1024, which is approximately 0.128.

    Rate this question:

  • 26. 

    The membership of a committee consists of 3 English teachers, 4 Mathematics teachers, and 2 Social Studies teachers.  If 2 committee members are to be selected at random to write the committee’s report, what is the probability that the two members selected will be an English teacher and a Mathematics teacher?  

    Correct Answer
    B.
    Explanation
    To find the probability of selecting an English teacher and a Mathematics teacher, we need to calculate the probability of selecting one English teacher and one Mathematics teacher, and then multiply these probabilities together.

    The probability of selecting an English teacher as the first member is 3/9, since there are 3 English teachers out of a total of 9 committee members.

    After selecting an English teacher, there are now 8 committee members left, including 4 Mathematics teachers. So the probability of selecting a Mathematics teacher as the second member is 4/8.

    Multiplying these probabilities together, we get (3/9) * (4/8) = 12/72 = 1/6.

    Therefore, the probability that the two members selected will be an English teacher and a Mathematics teacher is 1/6.

    Rate this question:

  • 27. 

    Two different pipes fill the same pool. Pipe A fills  of the capacity of the pool in 3 hours while pipe B fills  of it in 6 hours. If the two pipes fill the pool simultaneously with their respective rates how long will it take for the pool to be filled to capacity?  

    • A.

      3 hours

    • B.

      3 hours 20 minutes

    • C.

      3 hours 36 minutes

    • D.

      4 hours 20 minutes

    • E.

      4 hours 30 minutes

    Correct Answer
    C. 3 hours 36 minutes
    Explanation
    Pipe A fills the pool in 3 hours, which means it fills 1/3 of the pool's capacity per hour. Pipe B fills the pool in 6 hours, which means it fills 1/6 of the pool's capacity per hour. When both pipes are filling the pool simultaneously, their combined rate is 1/3 + 1/6 = 1/2 of the pool's capacity per hour. Therefore, it will take 2 hours for the pool to be filled to half of its capacity. Since the question asks for the pool to be filled to full capacity, it will take twice as long, which is 2 x 2 = 4 hours. Additionally, 1/2 hour is equal to 30 minutes, so the total time is 4 hours + 30 minutes = 4 hours 30 minutes.

    Rate this question:

  • 28. 

    What is the ratio of the average (arithmetic mean) height of students in class X to the average height of students in class Y?   (1)   The average height of the students in class X is 120 centimeters.   (2)   The average height of the students in class X and class Y combined is 126 centimeters.  

    • A.

      Statement (1) ALONE is sufficient, but statement (2) alone is not sufficient.

    • B.

      Statement (2) ALONE is sufficient, but statement (1) alone is not sufficient.

    • C.

      BOTH statements TOGETHER are sufficient, but NEITHER statement ALONE is sufficient.

    • D.

      EACH statement ALONE is sufficient.

    • E.

      Statements (1) and (2) TOGETHER are NOT sufficient.

    Correct Answer
    E. Statements (1) and (2) TOGETHER are NOT sufficient.
    Explanation
    The ratio of the average height of students in class X to the average height of students in class Y cannot be determined with the given information. Statement (1) tells us the average height of students in class X, but it does not provide any information about the average height of students in class Y. Statement (2) tells us the average height of students in both classes combined, but it does not provide any information about the individual averages of each class. Therefore, both statements together are not sufficient to determine the ratio.

    Rate this question:

  • 29. 

    Which of the following is equal to  ?

    Correct Answer
    A.
  • 30. 

    Pat invested x dollars in a fund that paid 8 percent annual interest, compounded annually.  Which of the following represents the value, in dollars, of Pat’s investment plus interest at the end of 5 years?

    • A.

      5(0.08x

    • B.

      5(1.08x)

    • C.

      [1 + 5(0.08)]x

    • D.

      (1.08)5x

    • E.

      (1.08x)5

    Correct Answer
    D. (1.08)5x
    Explanation
    The given expression, (1.08)5x, represents the value of Pat's investment plus interest at the end of 5 years. The expression is derived from the formula for compound interest, which is A = P(1 + r/n)^(nt), where A is the final amount, P is the principal amount (initial investment), r is the annual interest rate, n is the number of times the interest is compounded per year, and t is the number of years. In this case, the principal amount is x, the annual interest rate is 8% (or 0.08), and the interest is compounded annually (n = 1). Therefore, the correct answer is (1.08)5x.

    Rate this question:

  • 31. 

    In parallelogram PQRS shown, if PQ = 4 and QR = 6, what is the area of PQRS?  

    Correct Answer
    B.
    Explanation
    To find the area of a parallelogram, we need to multiply the base by the height. In this case, the base is PQ and the height is QR. Therefore, the area of PQRS is 4 * 6 = 24.

    Rate this question:

  • 32. 

    What is the remainder when the positive integer n is divided by 3?   (1)   The remainder when n is divided by 2 is 1.   (2)   The remainder when n + 1 is divided by 3 is 2.  

    • A.

      Statement (1) ALONE is sufficient, but statement (2) alone is not sufficient.

    • B.

      Statement (2) ALONE is sufficient, but statement (1) alone is not sufficient.

    • C.

      BOTH statements TOGETHER are sufficient, but NEITHER statement ALONE is sufficient.

    • D.

      EACH statement ALONE is sufficient.

    • E.

      Statements (1) and (2) TOGETHER are NOT sufficient.

    Correct Answer
    B. Statement (2) ALONE is sufficient, but statement (1) alone is not sufficient.
    Explanation
    From statement (1), we know that the remainder when n is divided by 2 is 1. This means that n is an odd number. However, we don't have any information about the remainder when n is divided by 3 from this statement.

    From statement (2), we know that the remainder when n + 1 is divided by 3 is 2. This means that (n + 1) is 2 more than a multiple of 3. Simplifying this, we get n is 1 less than a multiple of 3. In other words, n is 2 more than a multiple of 3. This means that the remainder when n is divided by 3 is 2.

    Therefore, statement (2) alone is sufficient to determine the remainder when n is divided by 3, but statement (1) alone is not sufficient.

    Rate this question:

  • 33. 

    If r, s, and t are positive integers, is r + s + t even?   (1)   r + s is even.   (2)   s + t is even.  

    • A.

      Statement (1) ALONE is sufficient, but statement (2) alone is not sufficient.

    • B.

      Statement (2) ALONE is sufficient, but statement (1) alone is not sufficient.

    • C.

      BOTH statements TOGETHER are sufficient, but NEITHER statement ALONE is sufficient.

    • D.

      EACH statement ALONE is sufficient.

    • E.

      Statements (1) and (2) TOGETHER are NOT sufficient.

    Correct Answer
    E. Statements (1) and (2) TOGETHER are NOT sufficient.
    Explanation
    The given question asks whether the sum of three positive integers, r + s + t, is even.

    Statement (1) tells us that r + s is even. However, this does not provide any information about the parity of t. It is possible that t is odd, which would make the sum r + s + t odd. Therefore, statement (1) alone is not sufficient to determine whether r + s + t is even.

    Statement (2) tells us that s + t is even. However, this does not provide any information about the parity of r. It is possible that r is odd, which would make the sum r + s + t odd. Therefore, statement (2) alone is not sufficient to determine whether r + s + t is even.

    When we consider both statements together, we still do not have enough information to determine the parity of r + s + t. It is possible that r is odd and t is even, or vice versa, which would make the sum r + s + t odd. Therefore, statements (1) and (2) together are not sufficient to determine whether r + s + t is even.

    Hence, the correct answer is that statements (1) and (2) together are not sufficient.

    Rate this question:

  • 34. 

    Of the families in City X in 1994, 40 percent owned a personal computer.  The number of families in City X owning a computer in 1998 was 30 percent greater than it was in 1994, and the total number of families in City X was 4 percent greater in 1998 than it was in 1994.  What percent of the families in City X owned a personal computer in 1998?

    • A.

      50%

    • B.

      52%

    • C.

      56%

    • D.

      70%

    • E.

      74%

    Correct Answer
    A. 50%
    Explanation
    In 1994, 40% of the families in City X owned a personal computer. In 1998, the number of families owning a computer increased by 30% compared to 1994. Additionally, the total number of families in City X increased by 4% in 1998 compared to 1994. Therefore, the percentage of families owning a personal computer in 1998 can be calculated by adding the 30% increase to the initial 40% ownership rate. This gives us a total of 70% ownership. However, since the total number of families also increased by 4%, the percentage of families owning a personal computer in 1998 will be slightly less than 70%. The closest option is 50%, which is the correct answer.

    Rate this question:

  • 35. 

    The operation ○ is defined by the equation = where y &ne -x.  If   then  =

    Correct Answer
    D.
  • 36. 

    Correct Answer
    D.
  • 37. 

    What is the value of   ?   (1)      (2)     

    • A.

      Statement (1) ALONE is sufficient, but statement (2) alone is not sufficient.

    • B.

      Statement (2) ALONE is sufficient, but statement (1) alone is not sufficient.

    • C.

      BOTH statements TOGETHER are sufficient, but NEITHER statement ALONE is sufficient.

    • D.

      EACH statement ALONE is sufficient.

    • E.

      Statements (1) and (2) TOGETHER are NOT sufficient.

    Correct Answer
    B. Statement (2) ALONE is sufficient, but statement (1) alone is not sufficient.
  • 38. 

    If x and y are integers, is x + y greater than 0?   (1)   x is greater than 0.   (2)   y is less than 1.  

    • A.

      Statement (1) ALONE is sufficient, but statement (2) alone is not sufficient.

    • B.

      Statement (2) ALONE is sufficient, but statement (1) alone is not sufficient.

    • C.

      BOTH statements TOGETHER are sufficient, but NEITHER statement ALONE is sufficient.

    • D.

      EACH statement ALONE is sufficient.

    • E.

      Statements (1) and (2) TOGETHER are NOT sufficient.

    Correct Answer
    E. Statements (1) and (2) TOGETHER are NOT sufficient.
    Explanation
    From statement (1), we know that x is greater than 0. However, this does not provide any information about the value of y. Therefore, statement (1) alone is not sufficient to determine whether x + y is greater than 0.

    From statement (2), we know that y is less than 1. However, this does not provide any information about the value of x. Therefore, statement (2) alone is not sufficient to determine whether x + y is greater than 0.

    Combining both statements, we still do not have enough information to determine whether x + y is greater than 0. Therefore, both statements together are not sufficient.

    Rate this question:

  • 39. 

    If the symbol ▼ represents either addition, subtraction, multiplication, or division, what is the value of 6 ▼ 2?   (1)   10 ▼ 5 = 2   (2)    4 ▼ 2 = 2  

    • A.

      Statement (1) ALONE is sufficient, but statement (2) alone is not sufficient.

    • B.

      Statement (2) ALONE is sufficient, but statement (1) alone is not sufficient.

    • C.

      BOTH statements TOGETHER are sufficient, but NEITHER statement ALONE is sufficient.

    • D.

      EACH statement ALONE is sufficient.

    • E.

      Statements (1) and (2) TOGETHER are NOT sufficient.

    Correct Answer
    A. Statement (1) ALONE is sufficient, but statement (2) alone is not sufficient.
    Explanation
    From statement (1), we know that 10 ▼ 5 = 2. However, we don't know the specific operation represented by ▼. Therefore, we cannot determine the value of 6 ▼ 2 based on statement (1) alone.

    From statement (2), we know that 4 ▼ 2 = 2. Again, we don't know the specific operation represented by ▼. Therefore, we cannot determine the value of 6 ▼ 2 based on statement (2) alone.

    When we consider both statements together, we still don't have enough information to determine the specific operation represented by ▼. Therefore, the value of 6 ▼ 2 cannot be determined.

    Hence, statement (1) alone is sufficient, but statement (2) alone is not sufficient.

    Rate this question:

  • 40. 

     In the figure shown what is the area of the circular region with center O and diameter BC? (1)      (2)      

    • A.

      Statement (1) ALONE is sufficient, but statement (2) alone is not sufficient.

    • B.

      Statement (2) ALONE is sufficient, but statement (1) alone is not sufficient.

    • C.

      BOTH statements TOGETHER are sufficient, but NEITHER statement ALONE is sufficient.

    • D.

      EACH statement ALONE is sufficient.

    • E.

      Statements (1) and (2) TOGETHER are NOT sufficient.

    Correct Answer
    C. BOTH statements TOGETHER are sufficient, but NEITHER statement ALONE is sufficient.
  • 41. 

     In the xy-plane line l and line k intersect at the point .  What is the slope of line l? (1)  The product of the slopes of line l and line k is &ndash1.   (2)    Line k passes through the origin.  

    • A.

      Statement (1) ALONE is sufficient, but statement (2) alone is not sufficient.

    • B.

      Statement (2) ALONE is sufficient, but statement (1) alone is not sufficient.

    • C.

      BOTH statements TOGETHER are sufficient, but NEITHER statement ALONE is sufficient.

    • D.

      EACH statement ALONE is sufficient.

    • E.

      Statements (1) and (2) TOGETHER are NOT sufficient.

    Correct Answer
    C. BOTH statements TOGETHER are sufficient, but NEITHER statement ALONE is sufficient.
    Explanation
    The slope of line l can be determined by using both statements together. Statement (1) tells us that the product of the slopes of line l and line k is -1. This means that the slopes of the two lines are negative reciprocals of each other. Statement (2) tells us that line k passes through the origin. Since line k passes through the origin, its y-intercept is 0, which means its slope is also 0. Therefore, the slope of line l can be determined by finding the negative reciprocal of 0, which is undefined. Thus, both statements together are sufficient to determine the slope of line l.

    Rate this question:

  • 42. 

    The infinite sequence a1, a2,…, an,… is such that a1 = 2, a2 = -3, a3 = 5, a4 = -1, and an = an-4 for n > 4.  What is the sum of the first 97 terms of the sequence?

    • A.

      72

    • B.

      74

    • C.

      75

    • D.

      78

    • E.

      80

    Correct Answer
    B. 74
    Explanation
    The given sequence follows a pattern where each term is equal to the term four places before it. By examining the first few terms of the sequence, we can see that it repeats every four terms: 2, -3, 5, -1, 2, -3, 5, -1, ... Therefore, we can group the terms into sets of four: (2, -3, 5, -1), (2, -3, 5, -1), ... The sum of each set of four terms is 3, so we can calculate the number of sets in 97 terms by dividing 97 by 4, which gives us 24 with a remainder of 1. Therefore, the sum of the first 97 terms is 24 * 3 + 2 = 74.

    Rate this question:

  • 43. 

    What is the value of │x + 7│? (1)  │x + 3│= 14   (2)    (x + 2)2 = 169  

    • A.

      Statement (1) ALONE is sufficient, but statement (2) alone is not sufficient.

    • B.

      Statement (2) ALONE is sufficient, but statement (1) alone is not sufficient.

    • C.

      BOTH statements TOGETHER are sufficient, but NEITHER statement ALONE is sufficient.

    • D.

      EACH statement ALONE is sufficient.

    • E.

      Statements (1) and (2) TOGETHER are NOT sufficient.

    Correct Answer
    C. BOTH statements TOGETHER are sufficient, but NEITHER statement ALONE is sufficient.
    Explanation
    Statement (1) gives the absolute value of (x + 3) as 14. This means that (x + 3) can either be 14 or -14. Therefore, x can be either 11 or -17. However, this does not give us the value of │x + 7│ directly.

    Statement (2) gives us (x + 2)² = 169. Taking the square root of both sides, we get x + 2 = ±13. This means that x can be either 11 or -15. However, this also does not give us the value of │x + 7│ directly.

    By combining both statements, we can narrow down the possible values of x to 11. Therefore, we can determine that the value of │x + 7│ is 18.

    Rate this question:

  • 44. 

    A student’s average (arithmetic mean) test score on 4 tests is 78.  What must be the student’s score on a 5th test for the student’s average score on the 5 tests to be 80?

    • A.

      80

    • B.

      82

    • C.

      84

    • D.

      86

    • E.

      88

    Correct Answer
    E. 88
    Explanation
    To find the student's score on the 5th test, we can use the formula for average: (sum of all scores) / (number of tests). The sum of the scores on the first 4 tests is 78 * 4 = 312. To have an average score of 80 on 5 tests, the sum of all 5 scores must be 80 * 5 = 400. Therefore, the student's score on the 5th test must be 400 - 312 = 88.

    Rate this question:

  • 45. 

    For a recent play performance, the ticket prices were $25 per adult and $15 per child.  A total of 500 tickets were sold for the performance.  How many of the tickets sold were for adults? (1)   Revenue from ticket sales for this performance totaled $10,500.   (2)   The average (arithmetic mean) price per ticket sold was $21.  

    • A.

      Statement (1) ALONE is sufficient, but statement (2) alone is not sufficient.

    • B.

      Statement (2) ALONE is sufficient, but statement (1) alone is not sufficient.

    • C.

      BOTH statements TOGETHER are sufficient, but NEITHER statement ALONE is sufficient.

    • D.

      EACH statement ALONE is sufficient.

    • E.

      Statements (1) and (2) TOGETHER are NOT sufficient.

    Correct Answer
    D. EACH statement ALONE is sufficient.
    Explanation
    Statement (1) alone is sufficient to determine the number of adult tickets sold. We can calculate the number of adult tickets by dividing the total revenue from ticket sales ($10,500) by the price per adult ticket ($25). This will give us the total number of tickets sold, and we can subtract the number of child tickets (which we don't know) from this total to find the number of adult tickets. Therefore, statement (1) alone is sufficient to answer the question.

    Rate this question:

  • 46. 

    A certain company assigns employees to offices in such a way that some of the offices can be empty and more than one employee can be assigned to an office.  In how many ways can the company assign 3 employees to 2 different offices?  

    • A.

      5

    • B.

      6

    • C.

      7

    • D.

      8

    • E.

      9

    Correct Answer
    D. 8
    Explanation
    The company can assign the employees in the following ways:
    1. All 3 employees in one office and the other office empty.
    2. 2 employees in one office and 1 in the other office.
    3. 1 employee in one office and 2 in the other office.
    4. All 3 employees in one office and the other office empty.
    5. 2 employees in one office and 1 in the other office.
    6. 1 employee in one office and 2 in the other office.
    7. All 3 employees in one office and the other office empty.
    8. 2 employees in one office and 1 in the other office.
    Therefore, there are 8 ways in which the company can assign 3 employees to 2 different offices.

    Rate this question:

  • 47. 

    If a code word is defined to be a sequence of different letters chosen from the 10 letters A, B, C, D, E, F, G, H, I, and J, what is the ratio of the number of 5-letter code words to the number of 4-letter code words?

    • A.

      5 to 4

    • B.

      3 to 2

    • C.

      2 to 1

    • D.

      5 to 1

    • E.

      6 to 1

    Correct Answer
    E. 6 to 1
  • 48. 

    What is the sum of a certain pair of consecutive odd integers? (1)   At least one of the integers is negative.   (2)   At least one of the integers is positive.  

    • A.

      Statement (1) ALONE is sufficient, but statement (2) alone is not sufficient.

    • B.

      Statement (2) ALONE is sufficient, but statement (1) alone is not sufficient.

    • C.

      BOTH statements TOGETHER are sufficient, but NEITHER statement ALONE is sufficient.

    • D.

      EACH statement ALONE is sufficient.

    • E.

      Statements (1) and (2) TOGETHER are NOT sufficient.

    Correct Answer
    C. BOTH statements TOGETHER are sufficient, but NEITHER statement ALONE is sufficient.
    Explanation
    Statement (1) tells us that at least one of the integers is negative. This means that the sum of the consecutive odd integers could be negative, depending on the specific values of the integers. Therefore, statement (1) alone is not sufficient to determine the sum of the integers.

    Statement (2) tells us that at least one of the integers is positive. This means that the sum of the consecutive odd integers could be positive, depending on the specific values of the integers. Therefore, statement (2) alone is not sufficient to determine the sum of the integers.

    However, when we consider both statements together, we know that one integer is negative and one integer is positive, which means their sum will always be an odd number. Therefore, both statements together are sufficient to determine the sum of the integers.

    Rate this question:

  • 49. 

    Is an odd integer? (1)    is an even integer.   (2)    is an odd integer.  

    • A.

      Statement (1) ALONE is sufficient, but statement (2) alone is not sufficient.

    • B.

      Statement (2) ALONE is sufficient, but statement (1) alone is not sufficient.

    • C.

      BOTH statements TOGETHER are sufficient, but NEITHER statement ALONE is sufficient.

    • D.

      EACH statement ALONE is sufficient.

    • E.

      Statements (1) and (2) TOGETHER are NOT sufficient.

    Correct Answer
    D. EACH statement ALONE is sufficient.
    Explanation
    The question asks whether the given integer is odd or not.

    Statement (1) alone tells us that the integer is even. Since an even integer cannot be odd, we can conclude that the given integer is not odd.

    Statement (2) alone tells us that the integer is odd. This directly answers the question and confirms that the given integer is indeed odd.

    Therefore, each statement alone is sufficient to determine whether the integer is odd or not.

    Rate this question:

  • 50. 

    Three printing presses, R, S, and T, working together at their respective constant rates, can do a certain printing job in 4 hours.  S and T, working together at their respective constant rates, can do the same job in 5 hours.  How many hours would it take R, working alone at its constant rate, to do the same job?

    • A.

      8

    • B.

      10

    • C.

      12

    • D.

      15

    • E.

      20

    Correct Answer
    E. 20
    Explanation
    If R, S, and T can complete the job together in 4 hours, it means that their combined rate is 1/4 of the job per hour. If S and T can complete the job together in 5 hours, their combined rate is 1/5 of the job per hour. Subtracting the combined rate of S and T from the combined rate of R, S, and T gives the rate of R alone, which is 1/4 - 1/5 = 1/20 of the job per hour. Therefore, it would take R 20 hours to complete the job alone.

    Rate this question:

Quiz Review Timeline +

Our quizzes are rigorously reviewed, monitored and continuously updated by our expert board to maintain accuracy, relevance, and timeliness.

  • Current Version
  • Feb 09, 2024
    Quiz Edited by
    ProProfs Editorial Team
  • Jun 18, 2012
    Quiz Created by
    Dakarazu

Related Topics

Back to Top Back to top
Advertisement
×

Wait!
Here's an interesting quiz for you.

We have other quizzes matching your interest.